Home
Current Affairs January 2024

What is the correct answer?

4

In males, which of the following structure(s) can NOT be palpated during digital rectal examination?

A. pelvic floor muscles

B. seminal vesicles

C. anterior surface of the sacrum

D. median lobe of the prostate

Correct Answer :

D. median lobe of the prostate


the median lobe, in particular, is engulfed by the lateral lobes and is not accessible for examination per rectum.

Related Questions

What is the correct answer?

4

Where is the gene sex-determining region (SRY) located at?

A. long arm of X chromosome

B. long arm of Y chromosome

C. short arm of Y chromosome

D. short arm of X chromosome

What is the correct answer?

4

Between which 2 fascial planes does the prostatic neurovascular bundle travel?

A. prostatic fascia and levator fascia

B. prostate capsule and prostatic fascia

C. Denonvilliers fascia and prostate capsule

D. Denonvilliers fascia and endopelvic fascia

What is the correct answer?

4

Which of the following conditions could result in white urine?

A. chyluria

B. bacteriuria

C. phosphaturia

D. all of the above

What is the correct answer?

4

Which statement is NOT related to Klinefelter syndrome?

A. sterility is the rule

B. has a considerable short life span

C. treatment requires testicular microdissection and sperm extraction

D. 47 XXY karyotype

What is the correct answer?

4

In males, which of the following structure(s) can NOT be palpated during digital rectal examination?

A. pelvic floor muscles

B. seminal vesicles

C. anterior surface of the sacrum

D. median lobe of the prostate

What is the correct answer?

4

Which of the following is false regarding zona glomeruloza?

A. the most superficial layer of the adrenal cortex

B. responses to increased potassium levels, renin or decreased renal blood flow

C. it causes pheochromocytoma

D. secretes aldosterone

What is the correct answer?

4

Which action of thigh muscles could be impaired if the obturator nerve was injured during endoscopic resection of bladder tumors?

A. adduction

B. abduction

C. flexion

D. extension

What is the correct answer?

4

What causes pure testicular feminization is:

A. over estrogen synthesis

B. defective end-organ androgen response

C. extra X chromosome (XXY)

D. failure of genital ridge regression

What is the correct answer?

4

Which statement is true about the female urethra?

A. harder to catheterize when compared to the male urethra

B. 4 inch long

C. opens into the vestibule below the clitoris

D. gets wider at the post-menopausal age

What is the correct answer?

4

What increases blood volume and pressure in corpora cavernosa during erection is:

A. increase in sympathetic discharge

B. contraction of ischiocavernosa and bulbocavernosa muscles

C. increase arterial flow and closure of emissary veins

D. shunting of blood from the dorsal to the central cavernosal artery

What is the correct answer?

4

Which part of the male urethra is the narrowest?

A. the membranous

B. the bladder neck

C. the bulbous

D. the external urethral meatus

What is the correct answer?

4

Which type of renal stone crystals is known as coffin lid?

A. oxalate

B. struvite

C. urate

D. cystine

What is the correct answer?

4

How many days does spermatogenesis take in the man?

A. 16

B. 32

C. 64

D. 90

What is the correct answer?

4

Where is Santorini plexus located?

A. at either side of the prostate

B. in the pubo-prostatic space

C. anterior to the seminal vesicles

D. posterior to the vaso-epididymal junction

What is the correct answer?

4

What is the creatinine clearance in ml/min for a 70 kg man aging 50 yrs. with 1.1 mg/dL serum creatinine?

A. 79.54

B. 44.19

C. 84.15

D. 94.25

What is the correct answer?

4

Which lab results indicate poor prognosis of male infertility?

A. azoospermia; normal FSH; increase LH

B. azoospermia; decrease FSH; normal LH

C. oligospermia; increase FSH; normal LH

D. oligospermia; normal FSH; decrease LH

What is the correct answer?

4

What antibiotic could result in red man syndrome?

A. vancomycin

B. clindamycin

C. streptomycin

D. tobramycin

What is the correct answer?

4

What is false about purple urine bag syndrome?

A. occurs due to the interaction between the chemical components of the urine and polyvinyl chloride material in the urine bag

B. it is commonly symptomless

C. more common in female nursing home residents

D. commonly implicated bacteria are K. pneumoniae, P. mirabilis, E. coli

What is the correct answer?

4

To which region does pain, from the left mid ureter, refer to?

A. the loin region

B. the inguinal region

C. the umbilical region

D. the penis or clitoris

What is the correct answer?

4

What is the nerve supply of the adrenal gland?

A. sympathetic to the medulla

B. parasympathetic to the medulla

C. sympathetic to the cortex

D. parasympathetic to the cortex

What is the correct answer?

4

What are the posterior relations of the kidneys?

A. quadratus lumborum and diaphragm

B. psoas and diaphragm

C. psoas and latissimus dorsi

D. transversus abdominus and paraspinous

What is the correct answer?

4

Which of the following is NOT related to von Hippel-Lindau disease?

A. retinal hemangioblastoma

B. ureteral atresia

C. pheochromocytomas

D. multiple cysts in the pancreas and kidneys

What is the correct answer?

4

What vessels are located in the prostate neurovascular bundle?

A. Watson plexus

B. capsular arteries and veins

C. inferior vesical and midrectal vessels

D. Santorini plexus

What is the correct answer?

4

What is true regarding hematospermia?

A. should be investigated thoroughly like hematuria

B. is rarely associated with significant urologic pathology

C. carries a risk of infertility

D. mandates testicular biopsy

What is the correct answer?

4

In which body tissue(s) are creatine and phosphocreatine converted to creatinine?

A. bone marrow

B. skeletal muscles

C. brown fat

D. liver

What is the correct answer?

4

Which class of the following diuretics could cause erectile dysfunction?

A. loop diuretics

B. thiazide diuretics

C. potassium sparing diuretics

D. osmotic diuretics

What is the correct answer?

4

What could render red urine workup, false positive for hematuria?

A. dehydration

B. myoglobinuria

C. high doses of vitamin C

D. all of the above

What is the correct answer?

4

The manifestations of autonomic dysreflexia include:

A. hypotension and tachycardia

B. hypotension and bradycardia

C. hypertension and tachycardia

D. hypertension and bradycardia

What is the correct answer?

4

When should a 70 yrs. man with 40 ml prostate volume, and total PSA of 2.0 ng/ml, go for a prostate biopsy?

A. if PSA reading had exceeded 2.7 ng/ml over 12 months

B. if free PSA reading was less than 0.4 ng/ml

C. if total PSA reading was greater than 8 ng/ml

D. any of the above

What is the correct answer?

4

How long post bilateral orchiectomy, will patients be functionally castrated?

A. immediately

B. after 2 hrs.

C. after 12 hrs.

D. after 24 hrs.